Step 1 Anatomy

Ace your homework & exams now with Quizwiz!

A 62-year-old male presents to your office with a painless mass in his groin. You are able to palpate several hard lymph nodes in the right inguinal area. Lymph node biopsy will reveal malignant cells which most likely originated from

*Anal Canal* Superficial inguinal lymph nodes - *anal canal* (below pectinate line), skin below umbilicus (except popliteal area), scrotum, vulva

Femoral hernias are more common in

*F*emales

An obstetrician performs a median episiotomy on a woman before parturition to prevent uncontrolled tearing. If the perineal body is damaged, the function of what muscles might be impaired?

*Bulbospongiosus & superficial transverse perineal* Perineal body / central tendon of perineum - fibromuscular node at center of perineum = attachment for bulbospongiosus, superficial & deep transverse perineal muscle, & sphincter ani externus muscles

58-year-old man diagnosed as having a slowly growing tumor in the deep perineal space. What structures would most likely be injured?

*Bulbourethral glands* Deep Perineal Space / Pouch contains - deep transverse perineal muscle & sphincter urethrae - membranous part of urethra - bulbourethral glands (in males) - branches of internal pudendal vessels & pudendal nerves

Elevated in Pancreatic cancer

*CA-19-9* Alkaline phosphatase GGT Amylase Lipase Total & Conjugated bilirubin

Provides major ligamentous support for the uterus

*Cardinal (Transverse Cervical) Ligament*

Recognize "apple core" lesion seen on barium enema x-ray

*Colorectal cancer*

An elderly man at a nursing home is known to have degenerative brain disease. When CSF is withdrawn by lumbar puncture for further exam, what structures is most likely penetrated by the needle?

*Ligamentum flavum* - CSF located in subarachnoid space = between arachnoid layer & pia mater Lumbar puncture: needle penetrates - skin, fascia, *ligamentum flavum*, epidural space, dura mater, subdural space, arachnoid mater

During surgical treatment of portal HT in a 59-year-old man with liver cirrhosis, a surgeon inadvertently lacerates dilated paraumbilical veins. The veins must be repaired to allow collateral flow. What ligaments is most likely severed?

*Ligamentum teres hepatis* Paraumbilical veins & ligamentum teres hepatic are contained in free margin of falciform ligament

A 31-year-old woman is brought to the ED after injuring her shoulder while playing soccer. Physical exam reveals an anterior dislocation of glenohumeral joint. The dislocation is reduced without complication. The patient returns because of significant, ongoing pain in the right arm & some weakness. She denies any sensory loss. Physical exam shows weakness of internal rotation of her right arm at the shoulder. An injury in what muscles is most likely cause of this patient's symptoms?

*Subscapularis* - arises from anterior surface of scapula --> inserts onto lesser tubercle of humerus - tendon passes anterior side of should joint capsule = reinforces capsule - contraction = *internal rotation* of arm at shoulder - *innervated by upper & lower subscapular nerves*

Rock climber falls on shoulder, resulting in chipping off of lesser tubercle of humerus. What structures would most likely have structural & functional damage?

*Subscapularis muscle* - inserts on lesser tubercle of humerus

A 16-year-old patient received a laceration of the posterior intercostal nerves by a penetrated knife blade. A pathologist obtained needle biopsy tissues & observed numerous degenerated cell bodies of the unipolar or pseudounipolar neurons. What structures would most likely provide the abnormal cell morphology?

*Dorsal root ganglion* - contains unipolar or pseudounipolar neurons Laceration of intercostal nerve injures... - GSE - postganglionic sympathetic GVE - GVA - GSA

Recognize *epidural fat* on MRI, which is where venous blood from internal venous plexus may spread

*E*

An elderly man with prostatitis is seen at the internal medicine clinic. The seminal colliculus of his prostate gland is infected, & its fine openings are closed. What structures is/are most likely to be disturbed?

*Ejaculatory ducts* - open onto seminal colliculus

A 12-year-old boy is brought to the ED by his parents because of a skateboarding accident which he fell on both outstretched hands. A normal radiograph of the hand is shown. The arrow shown in the normal x-ray of the hand depicts the bone that is most likely fractured in this patient. What is most likely to be seen in this patient as a result of the fracture?

*Elicited pain the anatomic snuffbox upon compression* Pain at anatomic snuffbox after fall on outstretched hands = pathognomonic for scaphoid fracture

23-year-old jockey falls from her horse & complains of headache, backache, & weakness. Radiologic exam would reveal blood in what spaces if internal vertebral plexus was ruptured?

*Epidural space* - space between vertebral canal & dura mater - contains internal venous plexus Internal Vertebral Venous Plexus - rout of early metastasis of carcinoma from lung, breast, & prostate gland to bones & CNS

36-year-old woman received a first-degree burn on her neck, arm, & forearm from a house fire. What skin structures or functions is most likely damaged or impaired?

*Exocrine gland secretion* Skin - sebaceous glands = exocrine glands

3-year-old boys admitted to children's hospital with complaints of restlessness, abdominal pain, & fever. An MI exam reveals that he has a double ureter. What embryonic structures is most likely failed to develop normally?

*Ureteric bud* - outgrowth of mesonephric duct - develops into ureter, renal pelvis, calyces, collecting tubules - bifurcated = partial duplication / bifid of ureter

53-year-old bank teller is admitted to a local hospital for a surgical removal of a benign pelvic tumor confined within the broad ligament. There is a risk of injuring what structures that lies in this ligament?

*Uterine tube* Broad ligament of uterus = 2 layers of peritoneum - holds uterus in position - contains uterine tube, uterine vessels, round ligament of uterus, ovarian ligament, lower part of ureter, uterovaginal nerve plexus, lymphatic vessels

V2 passes through what foramen?

*V2* *SRO = Standing Room Only* = V1-V3 *S*uperior orbital fissure - V1 Foramen *R*otundum - V2 Foramen *O*vale - V3

33-year-old man with perforated gastric ulcer complains of excruciating pain in his stomach. It is observed that the pain comes from the peritoneal irritation by gastric contents in the lesser sac. What nerves contain sensory nerve fibers that convey this sharp, stabbing pain?

*Vagus nerves* - carries sensory fibers associated with reflexes in GI tract - pain sensation originating from peritoneal irritation by gastric contents in lesser sac carried by lower intercostal nerves

An investigator is evaluating the function of the hypothalamic nuclei by ablating different parts of an experimental animal's hypothalamus & then monitoring the animal's behavior. In one experiment, after ablation, the animal begins to eat more food & becomes obese over a period of 2 weeks. What structure was most likely damaged in this experimental animal?

*Ventromedial nucleus* (VMN) = satiety center - bilateral destruction = hyperphagia, obesity, savage behavior - stimulation = urge to eat

3 arteries coming off the common hepatic artery

- *gastroduodenal* - right gastric artery --> follows lesser curvature of stomach --> anastomoses with left gastric artery - *proper hepatic* artery

The scrotum is innervated by branches of

- *ilioinguinal* - *genitofemoral* - pudendal - posterior femoral cutaneous

Additional findings expected in a man with Wernicke's (receptive) aphasia

- *right upper quadrantanopia* via lesion of optic radiations in temporal lobe - *fluent but nonsensical* - *word salad* via impairment in language comprehension - location in *temporal lobe* Wernicke's area in left (dominant) hemisphere = right upper quadrantanopia

A 54-year-old woman comes to the physician because of double vision. Neurologic exam reveals an inability to elevate the right eye from the abducted position. What additional findings would most likely be seen in this patient?

*A partial drooping of the right eyelid* Right superior rectus - innervated by oculomotor nerve --> also innervates levator palpebrae superioris = ptosis - *from abducted position, superior rectus is only muscle that can elevate eye* - superior rectus & inferior oblique can elevate eye from neutral position

A 4-month-old infant is evaluated for episodic partial gastrointestinal tract obstruction that has been present from birth. Esophagogastroduodenal endoscopic visualization fails to demonstrate an intraluminal lesion, but does show prominent narrowing at the level of the mid-duodenum. CT studies show a mass lesion surrounding the duodenum. What is the most likely underlying cause of these findings?

*Abnormal embryological rotation of the pancreatic bud* Annual pancreas - can cause early feeding difficulties - ventral bud rotates around right & left sides of duodenum - pancreatic head encircles second part of duodenum --> obstruction

32-year-old woman comes to the physician because of a mass in her left breast. A diagnosis of breast cancer is made & a mastectomy is planned. During the procedure, involvement of the muscle deep to the fatty tissue is seen & a radical mastectomy with removal of the breast, underlying chest muscles, & lymph nodes is performed. Removal of this muscle will most likely compromised what functions?

*Adduction of the humerus at the shoulder* Muscle deep to breast = pectoralis major - attaches to lateral lip of intertubercular groove of humerus = *adducts & internally rotates humerus at shoulder*

A 57-year-old homosexual man comes to the physician because of a mass near his groin. Physical exam shows a painless & enlarged superficial inguinal lymph node. A biopsy of the lymph node is performed under ultrasound guidance & shows the presence of squamous cell carcinoma. Metastasis from what sites has most likely occurred?

*Anal canal* - lymph nodes from groin divided into superficial inguinal & deep inguinal nodes *Tumors from penis, skin of labia, scrotum (not testes), & anal canal below dentate line drain to superficial group of inguinal lymph nodes* Tumor = squamous cell carcinoma = tumor arose from mucosa located below dentate (pectinate) line

78-year-old man suffering from ischemia of suprarenal glands. This condition results from rapid occlusion of direct branches of what arteries?

*Aorta, Inferior Phrenic, Renal arteries* - superior suprarenal artery via inferior phrenic artery - middle suprarenal artery via abdominal aorta - inferior suprarenal artery via renal artery

Presentations of median nerve injury

*Ape hand* - hand with thumb permanently extended Pope's blessing

A 67-year-old man comes to the physician becuse of hearing loss. Physical exam shows a large amount of wax blocking the patient's right ear canal. The patient faints as a water-filled syringe is used to expel wax from the ear canal. This event can be explained by the fact that the posterior half of the external ear canal receives innervation from

*Auricular branch of the vagus nerve* - since vagus also supplies many chest & abdominal organs, reflex symptoms may occur, including fainting (*vasovagal syncope*), coughing, gagging

A 34-year-old woman with a history of diabetes comes to the physician because of ear pain 2 days after swimming in a local pool. Physical exam shows a greenish discharge from the right ear canal. Erythema is localized to the anterior half of the external ear canal, & manipulation of the tragus elicits extreme tenderness. What nerves transmits the painful sensations this patient is experiencing?

*Auriculotemporal branch of the mandibular nerve* - external ear arises from first pharyngeal cleft

63-year-old man comes to ED with back pain, weakness, & shortness of breath. On exam, he has an aneurysm of abdominal aorta at the aortic hiatus of the diaphgragm. What pairs of structures would most likely be compressed?

*Azygoes vein & thoracic duct* Red = T12 Aorta (+ thoracic duct, azygos vein, greater splanchnic nerve) White = T10 Esophagus (+ vagus nerve) Blue = T8 IVC (+right phrenic nerve)

A 45-year-old man with a history of untreated hypercholesterolemia & HT is brought to the ED after collapsing at home. He never regains consciousness, & he dies 8 hours later. An autopsy shows a massive intracerebral hemorrhage filling the ventricles. What is most likely to be identified as the location of bleeding?

*Basal ganglia & internal capsule* Intraparenchymal hemorrhage - typically due to HT - most common cause of death from stroke - half of cases involve *basal ganglia & internal capsule* - Charcot-Bouchard aneurysms can develop & rupture in penerating lenticulostrite vessels of basal ganglia

17 yo boy injured in automobile accident. He has fracture of shaft of humerus. After accident, supination is still possible through contraction of what muscles?

*Biceps brachii* = flexor of elbow = strong supinator!!!

Floor of cubital fossa appears to be torn. What groups of muscles have lost their functions?

*Brachialis & supinator* = floor of cubital fossa

A 65 y/o male comes to see you in clinic with the chief complaint of right sided facial swelling & erythema. He is a 35 pack year smoker & has recently noticed a cough with random bouts of hemoptysis. Pt also complains of a 20 lb weight loss over the past year unintentionally. PE shows a cachetic with right sided JVD, engorged veins on RUE & face. What is most likely the source of his edema?

*Brachiocephalic Vein* / Innominate vein - formed by union of internal jugular vein & subclavian vein - at level of sternoclavicular joint - patient with SVC compression would produce engorgement on both sides of neck & face, not just the right side

During an investigational study, changes in heart rate & blood pressure with nerve stimulation are studied in experimental animals. The afferent portions of the glossopharyngeal & vagus nerves are selectively stimulated. What outocmes is most likely to occur during period of stimulation?

*Bradycardia with hypotension* Nucleus tractus solitarius (NTS) - artificially increasing firing rate of nerves --> NTS receives false signal that indicates BP is too high --> baroreceptor reflex = decrease in sympathetic nervous system outflow; increase in parasympathetic nervous system outflow --> bradycardia & hypotensin

During an investigational study, changes in heart rate & blood pressure with nerve stimulation are studied in experimental animals. The afferent portions of the glossopharyngeal & vagus nerves are selectively stimulated. What outcomes are most likely to occur during the period of stimulation?

*Bradycardia with hypotension* - CN IX & X carry afferent info to medulla from carotid sinus & aortic arch baroreceptors - nucleus tractus solitarius (NTS) receives input from these nerve

Examination of a 54-year-old man reveals an isolated tumor located at the porta hepatis. This tumor most likely compresses what structures?

*Branches of portal vein* Portal hepatis = transverse fissure (doorway) in liver = hepatic ducts, hepatic arteries, branches of portal vein

A 23-year-old man comes to the physician because of difficulty keeping liquid and saliva in his mouth. He reports that three months earlier, after a hiking trip in New Hampshire, he had developed flu-like symptoms & an expanding rash, but otherwise denies any other significant clinical history. he denies any loss of sensation or any vision or hearing problems. Physical exam shows facial asymmetry, flattening of labionasal fold, forehead wrinkles on one side, & saliva dripping from corner of his mouth when making chewing motions. Parlysis of what muscles is most likely in this patient?

*Buccinator* - innervated by facial nerve - holds food against teeth while it is chewed Facial nerve/Bell's palsy - via swelling of facial nerve within facial canal - via Lyme disease - paralysis of orbicularis oris = also innervated by facial nerve = allows accumulated food & saliva to drool from mouth

22-year-old man has a gonorrheal infection that has infiltrated space between the inferior fascia of the urogenital diaphragm & the superficial perineal fascia. What structure might be inflamed?

*Bulb of the penis* - located in superficial perineal space between inferior fascia of urogenital diaphragm & membranous layer of superficial perineal fascia (Colles fascia)

A 47-year-old Asian man comes to his physician because of 3-month history of weight loss & abdominal pain. He says that he has recently experienced vomiting & an overall feeling of malaise. His stools have also become darker recently. Lab studies show mild anemia but no evidence of infection. An abdominal CT scan shows a large mass protruding into the lumen of the stomach. The patient is given an initial diagnosis of gastric carcinoma & decides to undergo gastric carcinoma. In order to access the mass, the surgeon must dissect through the lesser omentum. What structures is most likely at risk for damage during the dissection of the lesser omentum?

*Common bile duct* Free edge of lesser omentum = hepatoduodenal ligament = portal triad - common bile duct - proper hepatic artery - portal vein

A 32-year-old woman with a history of Graves' disease has been refractory to medical therapy. The decision is made to perform a subtotal thyroidectomy. During the procedure the inferior thyroid artery is ligated; however, an adjacent nerve is inadvertently injured. Postoperatively, the patient complains of significant hoarseness. What muscles most likely retained it innervation after this injury?

*Cricothyroid* Recurrent laryngeal nerve = branch of vagus nerve - *innervates all intrinsic laryngeal muscles except the cricothryoid muscle* Cricothyroid muscle - innervated by external branch of superior laryngeal nerve - attached to cricoid cartilage & thyroid cartilage - contraction = stretched & adducts vocal ligament - inner branch of superior laryngeal nerve --> sensory innervation to mucosa of larynx above vocal folds

Recognize "string sign" on barium swallow x-ray

*Crohn disease* - any part of GI tract, but usually terminal ileum & colon - skip lesions - rectal sparing

A 27-year-old man who is an avid weightlifter comes to his physician because of a 6-month history of left arm pain. He has occasional shooting pain down the medial aspect of his left forearm & reports increasing difficulty with fine motor tasks. His past medical history is unremarkable, & he takes no medications. Physical exam reveals diminished sensation medially on the dorsal & palmar surface of his left hand. His left hypothenar eminence is flattened, while his thenar eminence appears normal. He is unable to keep his fingers spread apart against resistance. What is the most likely site of this patient's pathology?

*Cubital tunnel* Ulnar nerve entrapment - *risk factors = weight lifting, elbow trauma* Ulnar nerve - palmar & dorsal sensation to medial 1.5 digits - skin overlying hypothenar eminence - intrinsic hand muscles = dorsal & palmar interossei, third & fourth lumbrical muscles, flexor pollicis brevis, adductor pollicis, hypothenar muscles

Recognize pancrease on CT

*D = pancreas* A = kidney B = IVC C = portal vein E = descending colon

A 50-year-old man's x-ray of his right leg shows a fracture of the fibular head. What deficits would you most expect on physical exam?

*Decreased sensation on the dorsum of the right foot* *TIP: Tibilal Inverts & Plantarflexes* *PED: Peroneal Everts & Dorsiflexes* - common peroneal nerve = most commonly injured nerve in leg due to superficial location where it courses laterally around neck of fibula - sciatic nerve branches into common peroneal (fibular) nerve & tibilar nerve posteriorly on thigh just proximal to popliteal fossa - after coursing around neck of fibula, common peroneal nerve divides into superficial & deep branches - superficial branch innervtes muscles of lateral compartment of leg = functions to evert foot - deep peroneal nerve innervates anterior compartment of leg, whose muscles mainly acts as dorsiflexors of foot & toes - *superficial peroneal nerve gives of branches that provide sensory innervation to majority of dorsum of foot* - deep peroneal nerve provides sensory innervation only to region between first & second digits of foot = foot drop

43-year-old man has a benign tumor located near a gap between the arcuate pubic ligament and the transverse perineal ligament. What structures is most likely compressed by this tumor?

*Deep dorsal vein of the penis* - deep dorsal vein, dorsal artery, & dorsal nerve of penis pass through gap between arcuate ligamen & transverse perineal ligament

A 25-year-old man comes to the ED because of severe pain in the anterior portion of the left leg. He reports that earlier in the day during a football game, he suffered a blow to the shin from another player's helmet. He has noticed swelling of the leg & worsening pain over the last several hours. Physical exam shows significant weakness when attempting to extend the toes on the affected side but no issues with flexion of the toes. Dorsiflexion of the ankle is weak & elicits significant pain. Sensory exam reveals numbness on the dorsum of the foot on the left side of the web space between his first & second toe. Vascular exam shows absent dorsalis pedis pulse on the left side. Compression of what is most likely cause of this symptoms?

*Deep fibular (peroneal) nerve* Anterior compartment syndrome - via fluid accumulation in anterior fascial compartment of leg - *major nerve coursing through anterior compartment is deep fibular (peroneal) nerve* - *deep fibular (peroneal) nerve innervates tibialis anterior, major dorsiflexor of the foot, & also innervates the extensor digitorum longus, which extends all toes except the great toes & extensor hallucis long, the extensor of the great toe* - diminished dorsalis pedis pulse via compression of anterior tibial artery

37 yo female patient has fracture of clavicle. Lateral portion of fractured clavicle displaced downward by what?

*Deltoid muscle & gravity* - medial fragment is displaced upward by sternocleidomastoid - may damage subclavian vein --> pulmonary embolism

A 27-year-old man is involved in a motorcycle accident & sustains closed head injury. Physical exam shows periorbital ecchymosis & rhinorrhea with clear fluid. Otoscopic exam reveals blood behind the tympanic membrane. A CT scan of the head shows a basilar skull fracture in the region of the right foramen ovale. What is most likely to occur?

*Deviation of the mandible to the right upon protrusion of the mandible* Mandibular nerve passes through foramen ovale = sensory innervation of all muscles of mastication: masseter, temporalis, medial pteryhoid, lateral pterygoid muscles - lateral pterygoid muscle = protrusion of mandible = sensation in lower half of face

Innervate the deep muscles of the back, including the iliocostalis

*Dorsal primary rami*

19-year-old college student came to his doc's office for a neuro exam. His physician told him that normally synapses are absent in what structures?

*Dorsal root ganglia* = cell bodies of unipolar or pseudounipolar neurons - have no synapses

Symptoms of stroke with Anterior Cerebral Artery

- *contralateral lower limb paralysis & sensory loss* - medial aspect of primary motor cortex (lower limb & pelvis) supplied by anterior cerebral artery

A 49-year-old woman is brought to the ED 20 minutes after sustaining a closed head injury in a motor vehicle collision. A CT scan of the head shows a small tumor at the cerebellopontine angle of the brain but does not show any intracranial hemorrhage. What nerve is most likely to be affected by this tumor?

*Facial nerve* Cerebellopontine angle = triangular area formed by cerebellum, upper medulla, middle cerebellar peduncle - *facial nerve & vestibulocochlear nerve emerge from brain stem at cerebellopontine angle* - most common tumors in this area = acoustic schwannomas (neuromas) (80%) - remaining 20% = primarily meningiomas

Internal oblique abdominis muscle contributes to the formation of what structures?

*Falx inguinalis (conjoint tendon)* - formed by aponeuroses of internal oblique & transverse muscles of abdomen

A 52-year-old woman comes to the physician because of a 2-day history of pain in the groin. She also reports nausea & billious vomiting over the last 24 hours. Physical exam shows a tender bulge in the groin immediately inferior to the inguinal canal that is non-reducible. Auscultation of the abdomen reveals hyperactive, high-pitched bowel sounds. Lab studies shows slight acidosis with elevated lactic acid. What is located immediately lateral to site of this patient's pathology?

*Femoral vein* Femoral hernia - exits abdominal cavity via femoral ring to enter femoral canal, the medial region within femoral sheath - non-reducibility, bowel sounds suggestive of bowel obstruction, nausea/vomiting, elevated lactic acid = strangulation - more common in women - *emerge below inguinal ligament Femoral ring - bound laterally by femoral artery & vein (within femoral sheath) - *femoral vein = more medial vessel = immediately lateral to femoral ring* - inguinal ligament anteriorly - pubis posteriorly - lacunar ligament medially

A 52-year-old woman comes to the physician because of a 2-day history of pain in the groin. She also reports nausea & bilious vomiting over the last 24 hours. Physical exam shows a tender bulge in the groin immediately inferior to the inguinal canal that is non-reducible. Auscultation of the abdomen reveals hyperactive, high-pitched bowel sounds. Lab studies show slight acidosis with elevated lactic acid. What is located immediately lateral to the site of this patient's pathology?

*Femoral vein* = more medial = immediately lateral to femoral ring Femoral hernia - exits abdominal cavity via femoral ring to enter femoral canal, the medial region within the femoral sheath - more common women than in men - emerge below the inguinal ligament (while inguinal hernias emerge above inguinal ligament) Femoral ring bound - laterally = femoral artery & vein (within femoral sheath) - anteriorly = inguinal ligament - posteriorly = pubis - medially = lacunar ligament

42-year-old woman with metastatic breast cancer is known to have tumors in intervertebral foramina between fourth & fifth cervical vertebrae & between fourth & fifth thoracic vertebrae. What spinal nerves may be damaged?

*Fifth cervical & fourth thoracic nerves* - all cervical spinal nerves exit through intervertebral foramina above corresponding vertebrae, except eight cervical nerves, which runs inferior to 7th cervical vertebra - all other spinal nerves exit intervertebral foramina below corresponding vertebrae - fifth cervical nerve passes between fourth & fifth cervical vertebrae - fourth thoracic nerve runs between fourth & fifth thoracic vertebrae

39-year-old woman with headaches presents to her primary care physician with a possible herniated disk. Her MRI scan reveals that the posterolateral protrusion of the intervertebral disk between L4 & L5 vertebrae would most likely affect nerve roots of what spinal nerves?

*Fifth lumbar nerve* Posterolateral herniation of intervertebral disk at level L4 to L5 - affects fifth lumbar root but rarely affects fourth lumbar nerve root because of *progressive descending obliquity of fourth & fifth lumbar nerve roots*

A 51-year-old man who is a supermarket cashier comes to the physician because of weakness and numbness of his right hand. Physical exam shows loss of sensation over the palmar aspect of the thumb and the first two digits, & atrophy of the thenar eminence on the right hand. The nerve most likely injured also innervates what muscles?

*First lumbrical* Median nerve - supplies sensation to palmar aspect of thumb & adjacent two-and-one-half fingers - prior to entering carpal tunnel = supplies motor function to all *anterior forearm muscles except flexor carpi ulnaris* (ulnar nerve) & *one-half of flexor figitorum profundus* (ulnar nerve) - distal to carpal tunnel, median nerve also supplies *thenar muscles & first and second lumbricals*

A 66-year-old man is involved in a motor vehicle collision. Upon arrival, paramedics stabilize the patient's neck then transport him to the ED. He says that he has neck pain that radiates down the right arm. Physical exam shows a loss of sensation in a small region of the lateral upper right arm. An MRI of the spine shows a herniated intervertebral disc impinging on the right C5 nerve roots. What movements will be most likely affected?

*Flexion at the elbow* - C5 contributes to upper trunk --> contributes to lateral & posterior cord for anterior & posterior division of nerve fibers - C5 --> contributes to suprascapular nerve, musculocutaneous nerve, axillary nerve, lower subscapular nerve, upper subscapular nerve, lateral pectoral nerve, long thoracic nerve, part of phrenic nerve - *musculocutaneous nerve = major flexors of elbow*

A 61-year-old man who emigrated from Cambodia comes to ED because of fever, malaise, weight loss, cough, & recent back pain. Spinal x-rays show osteolytic bone lesions of the vertebral bodies in the lumbar region. After additional testing, a diagnosis of spinal tuberculosis is made. Two days after his initial visit, the patient develops pain in his right flank radiating to his groin. This patient's symptoms are most likely due to spread of infection along fascia of a muscle that attached to the affected vertebrae. This muscle most likely has what actions of the hip?

*Flexion* Psoas major - psoas abscesses occur after spinal infection --> spread along psoas major sheath; most common pathogen = S aureus; in endemic regions, Tuberculosis = cause often - attaches to trasverse processes & bodies of 12th thoracic vertebrae & all 5 lumbar vertebrae = spread of infection from bones into psoas major sheath is possible - principal action = *flex thigh at hip* - minimal action in external rotation of hip

Patient comes in complaining that she cannot flex her proximal interphalangeal joints. What muscles appear to be paralyzed on further exam of her finger?

*Flexor digitorum superficialis*

What landmark will aid a surgeon in distinguishing an indirect from a direct inguinal hernia?

*Inferior Epigastric Vessels* - *I*ndirect passes *l*ateral to *I*nferior epigastrics - *D*irect passes me*D*ial to inferior epigastrics MD = medial, direct

Left colic vein is a tributary of

*Inferior mesenteric vein*

A 42-year-old woman develops a painful blister over the vermillion of her upper lip. She has had several episodes of similar symptoms since age 12. The causal agent has most likely established latency on a nerve that exits the skull through what?

*Forum rotundum* HSV-1 - belongs to herpesvirus fam --> multiply in nucleus & surround themselves with nuclear membrane envelopes - can remain latent in *trigeminal ganglia* - treatment = acyclovir *S*uperior orbital fissure - V1 Foramen *R*otundum - V2 Foramen *O*vale - V3

Ligament that attaches greater curvature of stomach to transverse colon

*Gastrocolic ligament*

Young boy is brought to hospital after bicycle accident & possible pelvic fracture. While awaiting a CT scan of his pelvis, a physician proceeds with focal neuro exam. In testing the child's reflexes, what nerves would carry afferent impulses of cremasteric reflex?

*Genitofemoral nerve* Stimulation of cremasteric muscle - draws testis up from scrotum toward superficial inguinal ring - efferent limb of reflex arc = genital branch - afferent limb = femoral branch

A 9-year-old boy develops progressive difficulty walking & coordinating his limbs. The dysfunction spreads from the lower extremities to the arms & trunk. Physical exam show ataxic gait, nystagmus, & decreased deep tendon reflexes. Genetic testing shows a trinucleotide repeat expansion in the frataxin gene. Degeneration of cells in the dorsal root ganglia will most likely produce transsynaptic degeneration in what nuclei in this patient?

*Gracile & cuneate nuclei* Friedreich's ataxia (FA) = autosomal recessive neurodegenerative disease - associated with unstable trinucleotide repeat (GAA) in *frataxin* gene on chromosome 9 - progressive limb & gait ataxia, dysarthria, loss of proprioception & vibration sense, absent deep tendon reflexes in legs, & extensor plantar responses - onset generally between ages 8-15 - symptoms via *degeneration of large, myelinated sensory neurons in dorsal root ganglia (DRG)* - small, unmyelinated pain/temp fibers are spared - central axons of DRG neurons *ascend in poterior (dorsal) column within gracile or cuneate fasciculus* & project to their respective nuclei in the ipsilateral medulla

38-year-old woman with peptic ulcer disease of stomach experiences severe abdominal pain. What nervous structures is most likely involved?

*Greater splanchnic nerve* - carries pain fibers from upper GI tract

During a partial hysterectomy leaving the ovaries in tact, the surgeon detaches the ovary from the uterus by transecting the ovarian ligament. This ligament developed form what embryonic structure?

*Gubernaculum* - ovarian ligament & round ligament of uterus formed by gubernaculum

Fractured body of T4 vertebrae articulates with what part of the ribs?

*Head of fifth rib* - body of vertebra T4 articulates with heads of fourth & fifth ribs

A 43-year-old man comes to the ED because of a hand injury while golfing. He accidentally hit the ground very hard with his club while swinging causing immediate pain in his right hand. Exam shows marked tenderness of the palm near the fifth digit. Flexion of the fifth digit against resistance elicits pain at the same site. There is no reported loss of sensation in the hand. What is the most likely underlying cause of these findings?

*Hook of hamate fracture* - *ulnar nerve crosses wrist immediately lateral to hook of hamate & can be injured in such a fracture* - because *flexor digiti minimi & opponens digiti minimi have origins on hook of hamate* = flexion of fifth digit against resistance = pain at fracture site

A 43-year-old man comes to the ED because of a hand injury while golfing. He accidentally hit the ground very hard with his club while swinging causing immediate pain in his right hand. Exam shows marked tenderness of the palm near the fifth digit. Flexion of the fifth digit against resistance elicits pain at the same site. There is no reported loss of sensation in the hand. What is the most likely underlying cause of these findings?

*Hook of hamate fracture* - small bone in distal row of capral bones in wrist - sits proximal to little & ring fingers (5th & 4th digits) - *ulnar nerve crosses wrist immediately lateral to hook of hamate & can be injured in such a fracture* - *because flexor digiti minimi & opponens digiti minimi have origins on hook of hamate, flexion of fifth digit against resistance causes pain & fracture site* So Long to Pinky *Here* Comes The Thumb

19-year-old man with ruptured appendix is sent to ED for surgery. To cut off the blood supply to the appendix (if collateral circulation is discounted), a surgeon should ligate what arteries?

*Ileocolic artery* - appendicular artery = branch of ileocolic artery

During an appendectomy performed at McBurney point, what structures is most likely to be injured?

*Iliohypogastric nerve* - runs medially & inferiorly between internal oblique & transverse abdominal muscles near McBurney point

A 28-year-old woman delivers a boy at 42 weeks' gestation. She received minimal prenatal care during the pregnancy. A review of her records shows elevated levels of fetal alpha-fetoprotein but an ultrasound showed no neural tube defects. Physical exam of the newborn shows abdominal viscera, which are not covered by amnion, protruding through the abdominal wall adjacent to the midline. What is the most likely cause of these findings in the newborn?

*Incomplete fusion of the lateral folds* - 4th week of development = lateral body folds move ventrally & fuse in midline to form anterior body wall --> internalize dorsal part of yolk sac to form gut tube *Gastroschisis* - incomplete fusion of folds = defect in abdominal wall = allows abdominal viscera to herniate from abdominal cavity - viscera that protrude through defect are *not covered by amnionic membrane* covering the umbilical cord - usually occurs on right side of midline - elevated levels of alpha-fetoprotein prenatally

A standard urine test used to detect cocaine has a sensitivity of 97%, a specificity of 94%, a positive predictive value of 90%, and a negative predictive value of 95%. Recent data suggests that the use of cocaine in all age groups has declined since the screening test properties were determined. A decline in cocaine usage will most likely have what impact on the results of this screening test?

*Increased negative predictive value* Decreased positive predictive value

A 52-year-old man comes to the physician because of scrotal swelling. He also says that he has occasional urinary urgency & difficulty completely emptying his bladder. The patient is a former weightlifter, & acknowledges previous steroid use & occasional binge-drinking. Physical exam shows enlarged prostate & reducible scrotal mass. A few days later, he develops severe nausea, vomiting, & severe abdominal pain & is brought to the ED. His temp is 37.7 (99.8 F), pulse is 78/min, respirations are 14/min, & BP is 108/68 mmHg. Physical exam reveals distended abdomen & decreased bowel sounds in all 4 quadrants. The right scrotum is enlarged & edematous. A firm 6 cm mass is palpable with significant tenderness. Abdominal paracentesis is performed & fluid analysis reveals an elevated ammonia level. Gram stain shows gram-negative rods & cocci. what is most likely cause of these findings?

*Inguinal hernia strangulation* - although a hernia is reducible, it is at risk for incarceration & strangulation = painful to palpation - febrile - symptoms of bowel obstruction --> abdominal pain, decreased bowel sounds, vomiting, hypotension - peritoneal aspiration (paracentesis) = increased ammonia (via bacterial proliferation in damaged intestine) levels - gram stain of peritoneal fluid = enteric flora

5-year-old boy undergoes surgical repair of an indirect inguinal hernia. During surgery, a structure in one of the medial umbilical folds appears to be patent. What vessels directly gives rise to the patent structure?

*Internal iliac arteries* - medial umbilical fold = elevation of parietal peritoneum that covers medial umbilical ligament = adult remnant of umbilical artery - paired umbilical arteries arise from internal iliac arteries

A 60-year-old man who is a heavy smoker is referred to an urologist for gross hematuria. He denies any pain with urination or recent fevers. A cytoscopy shows a large mass projecting into the superior bladder. The patient undergoes surgical removal of the mass, which extends through the bladder wall & into the peritoneal cavity. Pathology reveals evidence of ischemic necrosis of the bladder wall. Compression of branches of what vessels will most likely cause this patient's ischemic necrosis?

*Internal iliac arteries* Bladder is supplied by: superior & inferior vesicle arteries - superior arteries = branches of umbilical arteries = branch of *internal iliac arteries* - inferior vesicle arteries = direct branches of internal iliac arteries

44-year-old woman comes to her physician & complains of headache & backache. On exam, she is found to have fluid accumulated in the spinal epidural space because of damage to blood vessels or meninges. What structures is most likely ruptured?

*Internal vertebral venous plexus* - located in spinal epidural space

34-year-old primigravid woman at 42 weeks' gestation is the second stage of labor. To provide anesthesia to this patient, an obstetrician plans a transvaginal pudendal nerve block. Through the lateral vaginal wall the obstetrician palpates a bony landmark & then inserts a needle toward that landmark to inject the anesthetic. What bony landmarks will most likely be used?

*Ischial spine* - pudendal nerve bock = local anesthetic near ischial spine

A 34-year-old primigravid woman at 42 weeks' gestation is in the second stage of labor. To provide anesthesia to this patient, an obstetrician plans a transvaginal pudendal nerve block. Through the lateral vaginal wall the obstetrician palpates a bony landmark & then inserts a needle toward that landmark to inject the anesthetic. What bony landmarks will most likely be used?

*Ischial spine* Pudendal nerve - can be anesthetized to provide obstetric anesthesia - arises from sacral segments 2, 3, 4 - exits pelvis via greater sciatic foramen --> courses around *ischial spine (& sacrospinous ligament)*& to pass thorugh lesser sciatic foramen - *ischial spine = boundary between greater & lesser sciatic foramina* - major branches of pudendal nerve = inferior rectal nerve, posterior labial (or scroatal nerve), & dorsal nerve of clitoris (or penis)

A 28-year-old primigravid woman comes to the ED following spontaneous rupture of membranes. She reports that she is now having periodic contractions about 5 minutes apart & they are increasing in frequency & intensity. The decision is made to perform an episiotomy to allow the baby to descend more easily & prevent soft tissue trauma. Prior to the procedure, a local anesthetic is administered to block sensation in that region. Palpation of what bony landmark would be most appropriate to locate the nerve that innervates this region?

*Ischial spine* Pudendal nerve - innervates perineum - originates from S2, S3, S4 - leaves pelvis via greater sciatic foramen --> *wraps around ischial spine* & sacrospinous ligament to enter lesser sciatic foramen --> eventually reaches perineum Ischial spine - separates greater sciatic foramen from lesser sciatic foramen - palpated by physician = located pudendal nerve in order to perform pudendal nerve block via transvaginal approach

A 43-year-old man comes to the physician because of pain in his groin. The patient reports previously noticing a bulge in the groin but only recently has it become painful. Physical exam reveals that Valsalva maneuver produces a bulge palpable in the superficial inguinal ring. Compression of the abdominal wall at the deep inguinal ring does not prevent the appearance of the bulge at the superficial ring. The physician recommends surgery to alleviate the patient's symptoms. What best describes the hernial sac seen in this patient?

*It passes media to inferior epigastric artery* *M*edial to inferior epigastric vessels = *D*irect hernia Direct inguinal hernia - protrudes through inguinal (Hesselbach) triangle - bulges through parietal peritoneum medial to inferior epigastric vessel but lateral to rectus abdominis - goes through external (superficial) inguinal ring only - covered by external spermatic fascia - usually in older men - via acquired weakness in transveralis fascia

26-year-old man's car was broadsided by a large truck, and he is brought to ED with multiple fractures of the transverse processes of the cervical & upper thoracic vertebrae. What muscles might be affected?

*Levator scapulae* - arises from transverse processes of upper cervical vertebrae - inserts on medial border of scapula

Symptoms of stroke with Lenticulostriate arteries

- *contralateral motor &/or of face &/or body* - *seen in HT patients* - hyaline arteriolosclerosis*

A 43-year-old man comes to the physician because of pain in his groin. The patient reports previously noticing a bulge in the groin but only recently has it become painful. Physical exam reveals that the Valsalva maneuver produces a bulge palpable in the superficial inguinal ring. Compression of the abominal wall at the deep inguinal ring does not prevent the appearance of the bulge at the superficial ring. The physician recommends surgery to alleviate the patient's symptoms. What best describes the hernial sac seen in this patient?

*It passes medial to the inferior epigastric artery* Direct inguinal hernia - enters inguinal canal by tearing through posterior wall of anal formed by transversalis fascia - through inguinal triangle - *direct inguinal hernias pass medial to inferior epigastric artery* - distinction between direct & indirect inguinal hernias is whether they pass through *deep inguinal ring (indirect) or do not pass through deep inguinal ring (direct)* - *if compression of deep ring does not prevent hernia from descending, then it is a direct inguinal hernia*

27-year-old woman involved in car accident is brought into ED. Her magnetic resonance imaging reveals that she has a laceration of the spinal cord at the L4 spinal cord level. What structures would you expect to be intact?

*Lateral horn* - in gray matter of spinal cord between *T1 & L2 and also between S2 & S4* - absent at L4!

A 70-year-old man with a history of diabetes & HT comes to the physician because of sudden-onset ataxia, hoarseness, & difficulty swallowing for the past 2 days. He says that he felt the room was spinning, then he fell to the ground & was unable to ambulate because of weakness & lack of coordination. One day later, he developed trouble swallowing food & hoarse voice. Neurologic exam shows anisocoria with left pupil constricted relative to right, left-sided ptosis, & decreased elevation of palate. Temperature & pinprick sensation are reduced on the left face & on the right side of the body below the neck. What diagram best represents the pattern of this man's lesion?

*Lateral medullary syndrome / Wallenberg syndrome* - via occlusion of *posterior inferior cerebellar artery (PICA)* - vestibular nuclei: vertigo, nausea/vomiting, nystagmus (away from lesion) - inferior cerebellar peduncle: ipsilateral limb ataxia - nucleus ambiguous (CN IX, X) = ipsilateral paralysis of larynx, pharynx, palate --> dysarthria, dysphagia, loss of gag reflex - spinothalamic tract = contralateral pain/temperature loss - descending hypothalamics = ipsilateral Horney syndrome (miosis, ptosis, anhydrosis)

27-year-old woman has suffered gunshot wound to her midabdomen. After examining the patient's angiogram, a trauma surgeon locates the source of bleeding from pairs of veins that typically terminate in the same vein. What veins are damaged?

*Left & right hepatic veins* - drain into the IVC

A 65-year-old man with a history of uncontrolled HT is brought to the ED after new onset weakness in his right leg. He denies any recent head trauma but does say that he has had a recent history of episodic palpitations & light headedness. Physical exam reveals 1/5 strength & decreased pinprick & light touch sensation in his right lower extremity. The patient is able to speak normally. Physical exam reveals no evidence of papilledema & pupils are equal, round, & reactive to light & accommodation. Occlusion of the following blood vessels is most likely responsible for this patient's symptoms?

*Left anterior cerebral artery* - patient had atrial fibrillation --> embolic stroke - HT = common predisposing factor for atrial fibrillation - *medial aspect of cerebral heisphere

A 64-year-old man is admitted to the hospital because of chest pain. An ECG reveals ST-segment elevation & T-wave inversion in V3-V4, & cardiac troponin T (cTNT) level is significantly elevated. Echocardiography shows a portion of the anterior-apical region of the left ventricle that bulges outward during systole & inward during diastole. What is the most likely location of the disease process?

*Left anterior descending artery* Anteroapical acute MI - *paradoxical movement* = when portion of ventricular wall is infarcted & can no longer contract during systole --> *think MI secondary to occlusion of LAD artery*

Radiograph of a 32-year-old woman reveals a perforation in the posterior wall of the stomach in which the gastric contents have spilled into the lesser sac. The general surgeon has opened the lienogastric (gastrosplenic) ligament to reach the lesser sac & notes erosion of the ulcer into an artery. What vessels is most likely involved?

*Left gastroepiploic artery* - runs through the lienogastric ligament

A 45-year-old man comes to the ED because of neck pain, dizziness, hoarseness, & facial numbness following chiropractic neck manipulation. Neurologic exam produces gag reflex with deviation of uvula to right when right side of pharyngeal mucosa is stimulated. When the left side of the pharyngeal mucosa is stimulated, the patient does not gag. These findings are most likely caused by a lesion in what structures?

*Left glossopharyngeal nerve & left vagus nerve* - gag reflex requires CN IX for sensory limb of reflex (unilateral) & CN X for moto limb of reflex (bilateral) - lesion of left CN IX will denervate sensory receptors on left side of pharynx --> when left side is touched, patient does not feel it & does not gag - If the left vagus nerve is lesioned, the left side of the soft palate will not elevate during gag & uvula will deviate to right

A 58-year-old man is brought to the ED because of weakness of the right arm & face & an inability to speak. He has a history of alcohol use, cigarette smoking, & HT. Neurologic exam shows an alert mental status. He is only able to say a few words with great effort, is unable to repeat, but can follow simply commands. He has a decreased right nasolabial fold & decreased movements of his right face, sparing the forehead. He has no right arm movement, but is able to raise his right leg. Occlusion of what blood vessels best explains this patient?

*Left middle cerebral artery* - supplies lateral surface of frontal, parietal, & upper temporal lobes; internal capsule; most of basal ganglia - broca aphasia, spastic paresis of contralateral lower face & upper limb, anesthesia of contralateral face & upper limb

Pediatric surgeon is resecting possible malignant mass from liver of a neonate with cerebral palsy. The surgeon divides roung ligament of liver during surgery. A fibrous remnant of what fetal vessels is severed?

*Left umbilical vein* - becomes round ligament after birth (ligamentum teres hepatis)

Woman is delivering a breech baby. The obstetrician decides that it is best to perform a mediolateral episiotomy. What structure should the obstetrician avoid incising?

*Levator ani* & External ani sphincter

43-year-old woman presents with prolapsed uterus. Repair of a prolapsed uterus requires knowledge of the supporting structures of the uterus. What structures plays the most important role in the support of the uterus?

*Levator ani* / Pelvic diaphragm

A 41-year-old woman comes to the physician because of a recent history of joint pain in her hands & knees, in addition to some mild weight loss & fatigue. She has a 2-week history of swelling behind her knee. She states that the swelling has been progressively growing, & she has pain & difficulty in flexing her knee. Her serum is positive for rheumatoid factor. Physical examination shows an erythematous, fluid-filled mass in the left lateral popliteal fossa overlying the tendon of biceps femoris. During aspiration of the cyst, a structure immediately adjacent to the tendon is injured. What losses is most likely to occur in this patient?

*Loss of dorsiflexion at the ankle* Baker's (popliteal) cyst - cycstic enlargement of popliteal bursa - filled with synovial fluid - can become inflamed & painful - often associated with rheumatoid arthritis Popliteal fossa: diamond shaped region behind kneed bounded by... - tendon of biceps femoris superolaterally - tendons of semimembranous & semitendinosus, superomedially - two heads of gastrocnemius, inferolaterally & inferomedially - in lateral part of fossa, *common fibular (peroneal) nerve lies immediately adjacent to tendon of biceps femoris* = might be injured in procedure - *common fibular nerve divides into superficial fibular & deep fibular nerves* --> *deep fibular nerve innervates muscles of anterior compartment of leg* (tibialis anteiror, extensor hallucis longus & extensor digitorum) = all *dorsiflexors of ankle*

An 11-year-old girl presents to her primary care physician after her seventh episode of a sore throat during the last year. Her mother reports that she has a history of recurrent upper respiratory tract infections. Because of this, the decision is made to undergo bilateral tonsillectomy. There is significant bleeding during the surgery & while attempting to ligate a blood vessel, the nerve that lies in the tonsillar fossa, deep to the palatine tonsil, is damaged. What deficits is most likely to result from this injury?

*Loss of sensation on the posterior one-third of the tongue* Nerve that lies in tonsillar fossa = glossopharyngeal nerve - *provides sensory innervation to mucosa of pharynx & general sensory & taste sensation to mucosa of posterior one-third of tongue*

A 50-year-old man who is an electrician comes to the physician because of recent onset pain & tingling in his left arm. Neurologic exam shows absence of sensation to light touch & pinprick on the fourth & fifth digits of his left hand & his medial forearm. There is also clawing of all digits of the left hand. Rotating the patient's head to the left side with the neck extended results in loss of the radial pulse on the left. An x-ray of the neck shows a cervical rib. What structures is most likely being compressed?

*Lower trunk of the brachial plexus* Thoracic outlet syndrome - lower trunk of brachial plexus & subclavian artery in scalene triangle --> pass laterally over upper surface of first rib & lie posterior to scalenus anterior - cervical rib, pancoast tumor - atrophy of intrinsic hand muscles, ischemia, pain, edema due to vascular compression

A 29-year-old man comes to the physician because of difficulty with fine motor control in his right hand. Two weeks ago, he fell from a ladder & as he was falling he reached out & grabbed the limb of a tree wit his right hand. Physical exam shows difficulty abducting & adducting the digits of the right hand, & inability to oppose the thumb on his right hand. What is most likely injured?

*Lower trunk of the brachial plexus* - nerve fibers from C8 & T1 - innervate intrinsic muscles of the hand = interosseous (abduction & adduction of digits) & opponense muscle (opposition of thumb) - contributes to formation of ulnar nerves & median nerves

Neurosurgeon performs a surgical resection of a rare meningeal tumor in the sacral region. He tries to avoid an injury of the nerve that arises from the lumbosacral plexus & remains within the abdominal or pelvic cavity. To what nerves should he pay particular attention?

*Lumbosacral trunk* - formed by part of ventral ramus of fourth lumbar nerve & ventral ramus of fifth lumbar nerve - *crosses the pelvic brim*

A newborn has projectile bilious vomiting shortly after each feeding. A review of his mother's records shows the presence of polyhydramnios during the pregnancy, but a karyotypic analysis was normal. What is the most likely cause of these findings in the newborn?

*Malrotation of the ventral pancreatic bud* - bilious vomiting = obstruction is distal to ampulla of Vater - both panreati buds form from evaginations from second part of duodenum = rotation around second part of duodenum distal to ampulla of Vater - normal rotation = around right side of embryonic duodenum Annular pancreas = bifid ventral pancreatic buds that rotates around both right & left sides of second part of duodenum --> stenosis of duodenum --> bilious vomiting - obstruction interferes with passage of amniotic fluid from mouth to jejunum prenatally = polyhydramnios

A newborn has projectile bilious vomiting shortly after each feeding. A review of his mother's records shows the presence of polyhydramnios during pregnancy, but a karyotypic analysis was normal. What is the most likely cause of these findings in the newborn?

*Malrotation of ventral pancreatic bud* - projectile vomiting after feeding = GI tract obstruction via stenosis or atresia - bilious vomiting = obstruction distal to ampulla of Vater - obstruction interferes with passage of amniotic fluid from mouth to jejunum prenatally polyhydramnios

A 34-year-old man who is a construction worker is brought to the ED after a recent fall at work. He says that he landed on his elbow & recently has had some weakness in his hand. Physical exam shows ecchymosis, swelling, & pain at the distal humerus & elbow. An initial neuro exam shows weakness with thumb adduction & sensory loss on the medial palm. There are no abnormalities with thumb abduction & flexion. Notably, there is clawing of the fourth & fifth digits. This patient most likely has an injury to a nerve that arises from what part of the brachial plexus?

*Medial cord only* Adductor pollicis - innervated by ulnar nerve, which arises from medial cord of brachal plexus - ulnar nerve also: innervates other intrinsic hand muscles & provides sensation to medial palm & medial 1.5 digits - ulnar nerve passes through cubital tunnel = prone to damage

Pediatric surgeon has resected a structure that is a fibrous remnant of an embryonic or fetal artery in a 5-year-old child. What structures is most likely to be divided?

*Medial umbilical fold* - contains fibrous remnant of *umbilical artery* - median umbilical fold contains fibrous remnant of urachus

35 yo man walks in with a stab wound to the most medial side of the proximal portion of the cubital fossa. What structures would most likely to be damaged?

*Median nerve* Contents of cubital fossa form medial to lateral side: median nerve brachial artery biceps brachii tendon radial nerve

A 19-year-old year old woman is brought to the ED after a recent snowboarding accident. The patient reports landing on her outstretched hand & now complains of wrist pain & swelling. She holds her hand in partial flexion due to pain on extension. Imaging of the wrist & hand reveals anterior dislocation of the lunate bone. What nerve is most likely to be compressed as a result of this injury?

*Median nerve* - only nerve the traverses *carpal tunnel* - innervates thenar eminence = flexion, abduction, opposition of thumb - innervates lateral two lumbricals = flexion of metacarpophalangeal joint & extension of interphalangeal joints of second & third digits - sensory innervation to palmar surface of lateral 3 1/2 digits as well as nail beds of these digits Lunate dislocation - typically involves *falling on outstretched & extended wrist* - one of four proximal carpal bone - displaced anterior into carpal tunnel - most likely to be dislocated of the carpal bones Carpal bones = *So Long To Pinky Here Comes The Thumb* - Proximal row, lateral-to-medial, then distal row, medial-to-lateral Scaphoid Lunate Triquetrum Pisiform Hamate Capitate Trapezoid Trapezium

A 1-week-old newborn is brought into the ED because of flluid draining from his umbilicus. Physical exam shows no abnormalities except a clear fluid that is pooled around the umbilical stump. Lab studies of the fluid shows a pH of 6, specific gravity of 1.020, 1-2 epithelial cells per high powered field, & no leukocytes, red blood cells, or hemoglobin. The fluid is passing through a patency of what adult remnant of an embryonic structure?

*Median* umbilical ligament

A 32-year-old primigravid woman in the first trimester is exposed to a person with mumps. The patient states that she has never been immunized, nor has she had mumps. The patient states that she has never been immunized, nor has sheen had mumps previously. Sever months later, she delivers a newborn with macrocephaly. A CT scan of the heads shows macrocephaly. A CT scan of the head shows hydrocephaly with symmetric enlargement of the lateral & third ventricles. The fourth ventricle is of normal size. What embryonic structures contributes to the development of the obstructed structure in this patient?

*Mesencephalon* Aqueductal stenosis = proximal ventricular enlagement & noncommunicating hydrocephalus - enlarged lateral & third ventricles & normal sized fourth ventricle - *cerebral aqueduct connects third to fourth ventricle* - cerebral aqueduct (of Sylvius) is in *midbrain or mesencephalon* - CSF flows from lateral ventricles through interventricular foramen of Monro to third ventricle, through cerebral aqueduct to fourth ventricle, out foramen of Magendie & foramina of Luschka into subarachnoid space, through arachnoid granulations, & out via dural venous sinuses & internal jugular vein

A 32-year-old primigravid woman in the first trimester is exposed to a person with mumps. The patient states that she has never been immunized, nor has she had mumps previously. Seven months later, she delivers a newborn with macrocephaly. A CT scan of the head shows hydrocephaly with symmetric enlargement of the lateral & third ventricles. The fourth ventricle is of normal size. What embryonic structures contribute to dev't of obstructed structure in this patient?

*Mesencephalon* Teratogenic result of mumps in first trimester = *aqueductal stenosis* - cerebral aqueduct connects third to fourth ventricle - cerebral aqueduct (of Sylvius) is in midbrain/mesencephalon

Temporalis muscle is innervated by a nerve originating from where?

*Mid-pons* - motor nucleus CN V - also supplies tensor tympani, tensor palati, mylohyoid, anterior belly of digastric

62 year old female responds with incoherent gibberish. She is unable to respond appropriately to questions or instructions but you notice she seems to be in high spirits. You suspect she may have suffered from a stroke, where is the likely culprit?

*Middle cerebral artery* - most common cerebral occlusion site - motor & sensory cortices - upper limb & face - temporal lobe (Wernicke area); frontal lobe (Broca area) - contralateral paralysis & sensory loss - face & upper limbs - aphasia if in dominant (left) hemisphere - hemineglect is lesion affects nondominant (right) side

A 35-year-old pregnant woman at 32 weeks' gestation comes to the physician because of pain and tingling in her hands. She also says that at times her hands feel numb, however she cannot pinpoint exactly where the numbness is centered. She reports no issues with her pregnancy. Physical exam shows wasting of the thenar eminence. Physical exam will most likely also reveal loss of sensation in what site?

*Nail bed of the third digit* Carpal tunnel syndrome - predispositions = hypothyroidism, pregnancy, amyloidosis, acromegaly, obesity, diabetes, work with vibrating tools - primary clue = *wasting of thenar eminence* - muscles of thenar eminence solely innervated by *recurrent branch of median nerve* - muscles = abductor pollicis brevis, flexor pollicis brevis, opponens pollicis --> control abduction, flexion, opposition of thumb - median nerve also innervates lateral two lumbrical muscles for digits 2 & 3 - median nerve supplies sensation to lateral two-thirds of palm, from thumb to lateral half of fourth (ring) finger = nerve also supplies distal dorsal surface of these fingers

58-year-old man admitted to hospital with severe abdominal pain, nausea, & vomiting resulting in dehydration. Emergency CT scan reveals a tumor located between celiac trunk & superior mesenteric artery. What structures is likely compressed by this tumor?

*Neck of pancreas* - pyloric canal & neck of pancreas situated anterior to abdominal aorta between origin of celiac trunk & superior mesenteric artery

During an exploratory laporatomy, a surgeon encounters significant bleeding in the region of the duodenum. In an effort to control the bleeding, the gastroduodenal artery is ligated. In the absence of the other arterial pathology, what is the most likely outcome?

*No ischemia of the stomach* - stomach has a *very rich anastomotic arterial supply* & can tolerate occlusion of the gastroduodenal artery

Innervates medial compartment of thigh, which mainly adduct the thigh at the hip

*Obturator nerve* - adductor longus, adductor brevis, anterior portion of adductor magnus, gracilis muscles - also provides *sensory innervation to skin on medial side of thigh*

A 22-year-old woman who is a flutist comes to the physician because of pain in her left forearm & tingling in her left hand. Physical exam shows decreased two-point discrimination on the palmar aspect of the 2nd & 3rd digits, & weakness of the 1st digit. Sensation over the thenar eminence remains intact. An electromyogram will most likely show reduced electrical activity in what muscles?

*Opponense pollicis* Carpal tunnel syndrome Median nerve, via recurrent branch, supplies: muscles of thenar eminence - opponense pollicis - abductor pollicis brevis - flexor pollicis brevis - lateral two lumbricals = prevent clawing of 2nd & 3rd digits

Function of Common Peroneal Nerve

*PED* *P*eroneal *E*verts & *D*orsiflexes injured --> foot drop*PED* - loss of sensation on dorsum of foot = L4-S2 Foot drop - inverted & plantarflexed at rest - loss of eversion & dorsiflexion - steppage gait

46-year-old woman has a history of infection in her perineal region. A comprehensive exam reveals a tear of the superior boundary of the superficial perineal space. What structures would most likely be injured?

*Perineal membrane* = inferior fascia of urogenital diaphragm = superior/deep boundary of superficial perineal space

What pairs of pharyngeal archest gives rise to arteries that contribute to the blood vessels that supply the brain?

*Pharyngeal arches 3 & 4* - each pharyngeal arch contains mesoderm - arch 3 = aortic arch that contributes to common carotid artery & internal carotid artery bilaterally - arch 4 = aortic arch that contributes to arch of aorta on left & right subclavian artery on right - internal carotid arteries branch into middle & anterior cerebral arteries that form anterior circulation of brain

A 29-year-old man comes to the physician because of tingling on the medial aspect of his left hand & forearm. Physical exam shows diminished radial pulse on the affected side & he undergoes decompression surgery to relieve the symptoms. During the procedure, the anterior scalene muscle is incised. What nerves is at greatest risk of injury during this procedure?

*Phrenic nerve* Thoracic outlet syndrome - thoracic outlet bordered by scalene muscles, first rib, clavicle - lower trunk of brachial plexus & subclavian artery are compressed between *anterior scalene muscle* & middle scalene muscle - incision of anterior scalene muscle relieves compression - *phrenic nerve* lies on anterior surface of anterior scalene muscle, deep to prevertebral fascia --> cut = diaphragm on that side of body will be paralyzed = numbness & pain on medial aspect of forearm & hand - weakness of muscles innervated by ulnar nerve in hand (claw hand) - decreased blood flow into upper limb = weakened radial pulse

Right colic vein is part of what venous system?

*Portal venous system* - empties into superior mesenteric vein, which joins splenic vein to form portal vein

17 yo boy with stab received multiple injuries on upper part of arm & required surgery. If brachical artery were ligated at its origin, what arteries would supply blood to profunda brachii artery?

*Posterior humeral circumflex* - anastomoses with ascending branch of profunda brachii artery

37-year-old small business manager receives a gunshot wound in the pelvic cavity, resulting in a lesion of the sacral splanchnic nerves. What nerve fibers would primarily be damaged?

*Preganglionic sympathetic fibers* --> come off chain & synapse in inferior hypogastric (pelvic) plexus Sacral splanchnic nerves = preganglionic sympathetic neurons & GVA fibers

In hypersecretion of NE from the suprarenal medulla, what type of nerve fiber is most likely overstimulated?

*Preganglionic* sympathetic fibers

While performing a pelvic exenteration, the surgical oncologist notices a fractured boundary of the pelvic inlet. What structure is most likely damaged?

*Promontory of the sacrum* Pelvic inlet / pelvic brim bounded by - promontory & anterior border of ala of sacrum - arcuate line of ilium - pectineal line - pubic crest - superior margin of pubic symphysis

A 51-year-old woman comes to the physician because of pain & numbness on the palmar surface of the first three digits of her left hand. She says that the symptoms are worse at night. Physical exam show reproducible pain & numbness of the left hand upon tapping the palmar surface of the lateral aspect of the wrist. The nerve that supplies the symptomatic region most likely passes through what muscles?

*Pronator teres* Carpal tunnel syndrome Median nerve - sensory innervation to skin of lateral portion of palmar surface of hand & to skin of palmar surface of first 3.5 digits - proximal to wrist, median nerve crosses elbow to enter forearm --> *passes between two heads of pronator teres muscle* = subject to entrapment neuropathy in this region

A 14-year-old boy is brought to the physician by his mother because of lumps in his breasts. He has been taking whey protein supplements because he would like to develop his muscles. Physical exam shows a 2-cm area of palpable tender breast tissue on the left & a 1-cm area on the right. There is no axiallary adenopathy. He denies recent trauma. What is most appropriate next step in patient care?

*Reassurance* Gynecomastia in puberty = common in adolescents - frequently asymmetric & tender - usually regresses before age 20 years - only reassurance typically needed

42-year-old man with portal hypertension secondary to cirrhosis of liver & subsequent massive ascites presents to ED. He refuses to have a transjugular intrahepatic portosystemic shunt (TIPS) procedure & prefers surgery. What surgical connections is involved in the most practical method of shunting portal blood around the liver?

*Splenic vein to the left renal vein* Portal HT can be reduced by diverting blood from portal to caval system = connecting splenic vein to left renal vein or by creating communication between portal vein & IVC

A 15-year-old girl is brought to the ED by her mother because of severe abdominal pain & a fever. The mother leaves the room before the exam, & the patient tells the physician she tried to terminate a pregnancy by inserting a sharp object in her vagina. Pelvic exam shows a small laceration of the posterior fornix of the vagina. Lab studies show leukocytosis & a serum pregnancy test is positive. Into what region did the sharp object most likely penetrate?

*Rectouterine space* - *posterior fornix is in contact with floor of rectouterine space* - rectouterine space = *pouch of Douglas* = lowest part of peritoneal cavity in female pelvis - in endometriosis, foci of endometrial tissue can be located in pouch of Douglas - in malignancy, peritoneal metastases can seed pouch of Douglas = mass that can be palpated on rectal exam (Blumer's shelf)

A 15-year-old girl is brought to the ED by her mother because of severe abdominal pain & fever. The mother leaves the room before exam, & the patient tells the physician she tried to terminate a pregnancy by inserting a sharp object into her vagina. Pelvic exam shows a small laceration of the posterior fornix of the vagina. Lab studies show leukocytosis & a serum pregnancy test is positive. Into what regions did the sharp object most likely penetrate?

*Rectouterine space* / Pouch of Douglas - posterior fornix in contact with floor of rectouterine space - lowest part of peritoneal cavity in female pelvis - located between uterus & rectum

A 40-year-old man with a history of alcoholism is brought to the ED because of recent onset coffee ground emesis. The patient also reports that his stools have recently become darker with a "tarry" consistency. He appears pale & his skin is cool & clammy; his pulse is 105/min & his blood pressure is 100/65 mmHg. Gastric lavage shows bright-red blood. Follow-up endoscopy reveals an ulcer with a visibly bleeding vessel located on the distal end of the lesser curvature of the stomach. The bleeding vessels are most likely direct branches of what arteries?

*Right gastric* - supplies distal portion of lesser curvature of stomach = branch of proper hepatic artery = anastomoses with left gastric artery (also supplies lesser curvature; direct branch of celiac trunk)

A 65-year-old man with a history of HT & diabetes is brought to the ED because of dysarthria & the inability to move his left arm & leg. His wife stages that he has smoked 1 pack of cigarettes daily for the past 35 years & he consumes 3-4 beers daily. He appears to be in some distress & has difficulty speaking. Physical exam shows an inability to move his left arm & leg, hyperreflexia on the left, & an upgoing plantar reflex on the left. He also has left facial weakness, sparing his forehead. Sensation in his face & extremities is intact. The lesion responsible for this patient's symptoms is most likely located in what areas?

*Right internal capsule* pure motor... - left-sided hemiplegia - left Babinski sign - left lower facial paralysis all on left must be in brain - facial paralysis + paralysis of body - lesion must be *contralateral* to signs & symptoms lesion must b above decussation/crossing = right sided since symptoms are left sided

A 66-year-old man with a history of hypertension is brought to the ED by his family after he is noticed to have behavioral changes. Neurologic exam shows that he is unable to repeat phrases spoken to him. What findings is most likely in this patient?

*Right upper quadrantanopia* Wernicke's (receptive) aphasia - produces speech that is *fluent but nonsensical* = word salad - temporal lobe in the posterior part of the superior temporal gyrus of the dominant (left) hemisphere - Wernicke's area often contains portion of optic radiation fibers from left lateral geniculate nucleus projecting to left primary visual cortex in occipital lobe (Meyer's loop) --> Wernicke aphasics commonly present with right upper visual field cut (*right upper quadrantanopia*) = upper quadrantanopia (pie in sky)

29 yo man comes in with stab wound, cannot raise his arm above horizontal, & exhibits "winged scapula." What structures of brachial plexus would most likely be damaged?

*Roots* - paralysis of serratus anterior = damaged to long thoracic nerve = arises form roots of C5-C7

26-year-old heavyweight boxer was punched on his mandible, resulting in slight subluxation (dislocation) of atlantoaxial joint. The consequence of injury was decreased range of motion at that joint. What movement would be most affected?

*Rotation* Atlantoaxial joints = synovial joints that consists of 2 plane joints & one pivot joint = rotation of head

22-year-old woman receives a deep cut in the inguinal canal 1 in. lateral to the pubic tubercle. What ligaments is lacerated within the inguinal canal?

*Round ligament of the uterus* - found in the inguinal canal along its course

Wrist bones

*S*o *L*ong *T*o *P*inky, *H*ere *C*omes *T*he *T*humb *S*caphoid *L*unate *T*riquetrum *P*isiform *H*amate *C*apitate *T*rapezoid *T*rapezium

An 84-year-old woman who lives in a nursing home has had multiple strokes. Following the last stroke, she has experienced worsening dysphagia & dysarthria. Neurologic examination shows an absent gag reflex. What is an additional function of the nerve that is involved in the afferent arc of this reflex?

*Salivation* Gag reflex - elicited by touching either side of posterior pharynx with tongue blade = bilateral elevation of palate & bilateral contraction of pharyngeal muscles - *afferent* of reflex arc = *ipsilateral glossopharyngeal nerve* - *vagus nerve, bilaterally* = efferent limb CN IX - *parotid glands* --> helps mediate salivation - taste & tactile sensation of posterior one-third of tongue = tactile sensation from external ear, pharynx, middle ear, auditory - input from carotid sinus/body - motor efferents to stylopharyngeus muscle - other salivary glands (submandibular & sublingual) = facial nerve

Chronic pain for past 5 years. Ongoing dull sensation with occasional sharp pain shooting down back of his leg. Difficulty climbing up stairs in his house. Difficulty extending the left thigh & flexing the knee,a s well as reduced sensation in left posterior thigh, lateral leg, and food. Compression of what nerve?

*Sciatic nerve* - has sciatica - arises from lumbosacral plexus via L1-S4 = tibial nerve (anterior L4-S3) + common fibular (peroneal) nerve (posterior L4-S2) = sciatic nerve

16-year-old boy presents to ED with rupture of penile urethra. Extravasated urine from this injury can spread into what structures?

*Scrotum*, Superficial perineal space, Penis, Anterior abdominal wall

5-month-old boy admitted to children's hospital because of urine being expelled from dorsal aspect of penis. What embryologic structures failed to fuse in this patient?

*Spongy urethra* - patient passes urine through opening on dorsum of penis

A 7-year-old boy is brought to the physician by his mother because of a mass on the side of his neck. Physical exam shows a small, mobile cyst along the anterior border of the sternocleidomastoid muscle. This cyst is the result of incomplete obliteration of what developmental structure?

*Second pharyngeal cleft* Normal development of neck... - second pharyngeal arch grows inferiorly = covers over second, third, & fourth pharyngeal clefts to fuse with cardiac ridge --> *obliterates clefts 2-4* - cervical sinus formed by fusion of these tissue masses is normally a transient structure but may persist as branchial or lateral cervical cyst = usually derived from second, third, or fourth pharyngeal cleft

A 45-year-old man with a history of vertigo is brought to the ED because of loss of consciousness after a fall. He appears mildly confused but is not in any acute distress. Physical exam of the head shows blood behind the tympanic membrane. A CT scan of the head shows a basilar skull fracture fragment compromising the foramen rotundum. What will most likely result form the patient's foramen rotundum injury?

*Sensory defects in the midface* - maxillary division of trigeminal nerve (V2) provides sensory innervation to midface - V2 transverses foramen rotundum --> damage = sensory abnormality in midface

A 63-year-old woman has difficulty elevating her left arm 2 days after a left-sided lumpectomy & axillary dissection for invasive ductal carcinoma of the breast. On physical exam, the patient is unable to fully raise her left arm from the side of her body. The medial border & inferior angle of left scapula become unusually prominent when she pushes against a wall with both hands. Damage to the nerves innervating what muscles is the most likely cause of these findings?

*Serratus anterior* - injury to *long thoracic nerve (C5-C7)*, which innervates serratus anterior muscle - paralysis of serratus anterior muscle = winged scapula = protrusion of scapula as patient pushes against wall

A 60-year-old man comes to the physician because of progressively worsening early satiety after meals for the past 4 months. He states that he has lost 13.6 kg (30 lb) over this time period. He has smoked 1 pack of cigarettes daily for the past 30 years. He has no history of any major medical illnesses and takes no medications. Physical exam shows a thin man and the spleen is palpated 5 cm below the costal margin. A CT scan of the abdomen shows a large gastric tumor impinging on the splenic artery as it passes posterior to the greater curvature of the stomach. Blood supply to the tissue supplied by what arteries will most likely be compromised?

*Short gastric* Splenic artery - passes behind stomach --> *gives off short gastric arteries* & *left gastro-omental artery* after passing greater curvature but before reaching spleen - short gastric arteries supply fundus of stomach

A 50-year-old man is brought to the ED because of loss of consciousness after an altercation with a fam member. His blood pressure is 90/60 mmHg & his pulse is irregular. Physical exam shows 1.5-cm stab wound at sternal border of right third intercostal space. What structures is most likely damaged as result of penetrating wound?

*Sinoatrial node* - in wall of right atrium near point of entry of SVC

A 45-year-old woman comes to the physician because of recent onset discharge of white milky fluid from her breast. She reports that her most recent mammogram & pregnancy tests were negative. She has also recently noticed significant morning headaches & indicates that her vision has gradually become more blurry in the last few months. She is afraid to drive because occasionally she is not able to see the cars in the lanes next to her. Physical examination shows bilateral papilledema, bilateral hemianopia, & no focal sensory or motor deficits. An MRI of the brain reveals a mass in the sella turcica & a hypophysectomy is planned. The tumor will most likely be resected transnasally through what structure?

*Sphenoid sinus* - located in sphenoid bone under pituitary gland - pituitary can be approaches transnasally via sphenoid sinus for hypophysectomy - bilateral hemianopia = vision is missing in outer (temporal) half of both right & left visual fields

After repair of a ruptured diverticulum, a 31-year-old patient begins to spike with fever & complains of abdominal pain. An infection in the deep perineal space would most likely damage what structures?

*Sphincter urethrae*

26-year-old man comes to hospital with fever, nausea, pain, & itching in the perineal region. On exam by a urologist, he is diagnosed as having infected bulbourethral (Cowper) glands. What structures is/are affected by this infection?

*Sphincter urethrae* Bulbourethral glands lie on either side of membranous urethra, embedded in sphincter urethrae - ducts open into bulbous part of penile urethra

42-year-old woman who has had six children develops weakness of the urogenital diaphragm. Paralysis of what muscles would cause such a symptom?

*Sphincter urethrae* Urogenital diaphragm = sphincter urethrae & deep transverse perineal muscles - weakness via multiple child delivery, advancing age, menopause

A 61-year-old man with a longstanding history of cigarette smoking & alcohol use is diagnosed with oral squamous cell carcinoma. He undergoes surgical resection of the lateral tongue & severe right-sided anterior cervical lymph nodes are also evaluated for the presence of canerous cells. Two days later, he returns to the physician because he is unable to raise his right arm above horizontal position and is unable to shrug his right shoulder. What nerve is most likely to have been injured?

*Spinal accessory nerve* - crosses posterior triangle of the neck - immediately deep to investing fascia of neck = at risk of injury in cervical lymph node dissection - *innervates trapezius* = upward rotation of scapula - also innervates sternocleidomastoid = rotates head

43-year-old woman admitted to hospital because of deep abdominal pain in her epigastric region. On exam, it is observed that a retroperitoneal infection erodes an artery that runs along superior border of pancreas. What artery is likely injured?

*Splenic artery* - arises from celiac trunk - runs along superior border of pancreas - enter spleen via lienorenal ligament

An investigator is isolating mucin-secreting cells from various tissues to compare surface receptor types on these cells. What tissues would most likely yield the highest percentage of mucin-secreting cells?

*Sublingual gland* Salivary glands = serous or mucous secreting cells - sublingual = mostly mucous - parotid = serous - submandibular = mixed serous & mucous

Retrograde urethrogram performed & leakage of dye indicates that there is a urethral tear at the point where the urethra enters the bulb of the penis. Into what spaces is the dye most likely leaking?

*Superficial pernieal space* Rupture of urethra at junction of *mebranous urethra* & *spongy urethra* - membranous portion passes through *urogenital diaphragm* - *bulb of penis is in superficial perineal space* - space is bounded by urogenital diaphragm above superficial pernieal (Colles') fascia below - other structures in this space (male) = crura of penis, ischiocavernosus muscles (covers crura of penis), bulbospongiosus muscle (covers bulb of penis) - if urine escapes this space, it can spread into anterior abdominal wall, into scrotum & along shaft of penis

A 26-year-old man comes to the physician because of a 3-week history of headaches & difficulty looking up. Neurologic exam shows pupils are 6 mm in diameter bilaterally with minimal reaction to light. Pupils do react to accomodation. He is unable to look upward past the horizontal plane, but otherwise has normal eye movements. MRI of the head shows a brain tumor. Compression of what structure is most likely responsible for the findings of this patient?

*Superior colliculus* Parinaud syndrome - via pineal tumor in this case - compression of superior colliculi & pretectal area of dorsal midbrain, region that contains vertical gaze center = vertical palsy Pineal tumors - compress cerebral aqueduct = noncommunicating hydrocephalus - large pupils that don't react to light but do react to near-far acommodation = light-near dissociation = accommodate but do not react - light-near dissociation also occurs with neurosyphilis (Argyll Robertson pupil) & diabetes mellitus

54-year-old man comes to hospital with abdominal pain, jaundice, loss of appetite, & weight loss. On exam of his radiograms & CT scans, a physician finds a slowly growing tumor in the uncinate process of the pancreas. What structures is most likely compressed by this tumor?

*Superior mesenteric artery* Uncinate process = projection of lower part of head to left behind superior mesenteric vessels

78-year-old man has malignancy in third part of duodenum. What structure is at greatest risk for direct penetration by tumor?

*Superior mesenteric artery* duodenum = shortest & thickest - third part = horizontal = top of L4 - *superior mesenteric artery & vein come down from underneath the pancreas to over the third or horizontal section*

29-year-old woman with abdominal pain was admitted to local hospital, & exam shows that a retroperitoneal infection is affecting a purely endocrine gland. What structure is infected?

*Suprarenal gland* - retroperitoneal - purely endocrine

A 53-year-old man comes to the physician because of worsening morning headaches. An MRI of the brain shows an invasive glioma at the base of the brain. He undergoes surgical resection of the tumor. However, cranial nerves IX & X are accidentally severed bilaterally during the procedure. What will most likely be seen immediately in this patient?

*Tachycardia with hypertension* - severing nerves sends medulla false signal that patient has suddenly lost all blood pressure - elicits baroreceptor reflex --> increase in sympathetic outflow & leading tachycardia & HT

31 yo roofer walks in with tenosynovitis resulting from deep wound in palm by big nail. Exam indicates that he has infection in ulnar bursa. This infection most likely resulted in necrosis of what tendons?

*Tendon of flexor digitorum profundus* Ulnar bursa / Common synovial flexor sheath - contains tendons of both flexor digitorum superficialis & profundus muscles

42-year-old obese woman with seven children is brought to a local hospital by her daughter. Physical exam & her radiograph reveal that large gallstones have ulcerated through posterior wall of fundus of gallbladder into the intestine. What parts of the intestine is most likely to initially contain gallstones?

*Transverse colon* - fundus of gallbladder is in contact with transverse colon --> gallstones erode through posterior wall of gallbladder & enter transverse colon

Recognize "lead pipe" appearing on imagind

*Ulcerative Colitis* - via loss of haustra

Man injures his wrist on broken glass. What structures entering the palm superficial to the flexor retinaculum may be damaged?

*Ulnar nerve & ulnar artery* Structures entering palm superficial to flexor retinaculum = ulnar nerve, ulnar artery, palmaris longus tendon, palmar cutaneous branch of median nerve

Man unable to hold typing paper between his index & middle fingers. What nerves was likely injured?

*Ulnar nerve* To hold typing paper, index finger adducted by palmar interosseous muscle, & middle finger abducted by dorsal interosseous muscle - both muscles innervated by ulnar nerve

21 yo man injures his right arm in accident. Radiographic exam reveals fracture of medial epicondyle of humerus. What nerves is most likely injured as result of this accident?

*Ulnar* nerve - runs down medial aspect of arm & behind medial epicondyle in groove - innervates *adductor pollicis muscle* - *unable to adduct index finger* = fingers adducted by palmar interosseous muscle; abduction via dorsal interosseous muscles

A 15-year-old boy is brought to the ED because of injuries to his head & right shoulder that he sustained after falling from his bicycle. The boy's right arm is hanging by his side with the palm facing posteriorly. He is unable to abduct the arm, has significant weakness of elbow flexion, and there is diminished sensation over the lateral aspects of the arm & forearm. An injury to what is most likely responsible for this patient's symptoms?

*Upper trunk of the brachial plexus* - contains nerve fibers from anterior rami of 5th & 6th cervical spinal nerves = Erb's palsy - deltoid & supraspinatus muscles affectes = responsible for abduction of arm at shoulder - all muscle responsible for external rotation at shoulder (teres minor, infraspinatus, & posterior portion of deltoid) are affected - waiter's tip sign - C5 & C6 contribute to innervation of biceps & brachialis muscles = weakness of flexion at elbow - sensory innervation of lateral side of arm & forearm is derived from 5th & 6th cervical roots

A 32-year-old man is brought to the ED after sustaining an injury to back of the left side of his neck during a physical altercation. A CT scan of the spine shows penetration by a bony fragment into the lateral portion of the dorsal columns. What functions is most likely to be affected by a lesion at this site?

*Vibratory sense from the left arm* At level of neck, lateral portion of the dorsal columns (funiculus) = comprised of fasciculus cuneatus - fasciculus cuneatus carries tactile, proprioceptive, vibratory info from arms = found in cervical & upper thoracic levels - fasciculus grascilis carries similar info from legs = present at all levels - internal arcuate fibers decussate in caudal medulla - lesions above decussation --> contralateral symptoms - lesions below decussation --> ipsilateral sympatons

A 24-year-old man who is a construction worker comes to the physician because of awkwardness in using his left hand. He reports that two months ago he fell on his outstretched hand & landed on the heel of the hand. Physical exam shows that the 4th & 5th fingers are extended at the metacarpophalangeal joints & flexed at the interphalangeal joints. What additional findings is most likely to be seen on physical exam?

*Wasting of the interosseous muscles* Ulnar nerve lesion - clawing of ring & little fingers - can also produce *wasting of hypothenar eminence & interoseeous muscles* = guttering between extensor tendons on back of hand - dorsal interosseous muscle abduct II-IV & palmar interosseous muscles adduct digits II-V - loss of sensation to medial side of hand (both palm & dorsal surface) & both palmar & dorsal surfaces of little finger & medial half of fourth finger

A study is conducted to chracterize embryonic hemoglobin synthesis. 32P-labeled cDNA probes for human zeta- & epsilon-mRNAs are used to localize areas in which embryonic hemoglobin is synthesized. Embryonic tissues from different tissues from aborted embryos are hybrizied with labeled probes. In what areas would the probes show the most robust hybridization in 3-week-old embryos?

*Yolk sac* Beginning at 1 month - 7th month = blood elements formed in liver Last month before birth = bone marrow becomes dominant site of hematopoiesis, as well as spleen & lymphatic organs

Major actions of pectoralis major, which attaches to lateral lip of intertubercular groov of humerus, is to

*adduct & internally rotate humerus at shoulder* = breast cancer can invade pectoralis major - radical mastectomy involves removal of pectoralis major

A 21-year-old marine biologist asks about her first bimanual exam, & it is explained to her that the normal position of the uterus is

*anteflexed & anteverted* Normal position of uterus: anteverted = angle of 90 degrees at junction of vagina & cervical canal anteflexed = angle of 160-170 degrees at junction of cervix & body

Spinal cord crushed at level of upper part of first lumbar vertebra. What structure is most likely damaged?

*conus medullaris* = conical end of spinal crod = terminates at level of L2 vertebrae or intervertebral disk between L1 & L2

Genitofemoral nerve passes through

*deep inguinal ring* Genitofemoral nerve - descends on anterior surface of psoas muscle --> gives rise to genital branch --> enters inguinal canal through deep inguinal ring to supply cremaster muscle - arises from lumbar plexus

While examining radiographs & angiograms of a 52-year-old patient, a physician is trying to distinguish jejunum from ileum. He has observed that the jejunum has

*fewer mesenteric arterial arcades* but longer vasa recta than ileum - more digestion & absorption of nutriends in jejunum - less fat in mesentary of jejunum

Anastomoses that effectively connects superior mesenteric artery with celiac trunk

*inferior & superior pancreaticoduodenal arteries*

Ischiorectal fossa contains

*inferior rectal nerves, vessels,* & adipose tissue

How does clavicle develop?

*intramembranous ossification* Clavicle = only upper limb bone to develop via intramembranous ossification = first upper limb bone to begin ossification = last bone to complete ossification = forms from somatic lateral plate mesoderm

Quadrate lobe of liver drains bile into

*left hepatic duct* Quadrate lobe of liver - receives blood from left hepatic artery - lies between gallbladder fossa & ligamentum teres hepatic - medial inferior segment - part of left lob

Serratus anterior is innervated by

*long thoracic nerve* - breast surgery --> injury to long thoracic --> paralysis of serratus anterior --> winged scapula = protrusion of scapula as patient presses against wall; cannot abduct arm above horizontal position

Supracondylar fracture of the humerus can damage what nerve?

*median nerve* - *ulnar deviation of hand on flexion at wrist can be seen via unopposed action of flexor carpi ulnaris*

Sphincter urethra innervated by

*perineal* branch of pudendal nerve Sphincter urethra = *striated* muscle - lies in deep perineal space - forms part of urogenital diaphragm

Radial artery divides into what at hand?

*princeps pollicis* & deep palmar arterial arch - stenosis of radial artery = decreased blood flow in princeps pollicis artery

Are of body drained by inferior mesenteric lymph nodes

*sigmoid colon* colon from splenic flexure to upper rectum

Area of body drained by Para-aortic lymph nodes

*testes*, ovaries, kidney, uterus

A 23-year-old man is brought to the ED by paramedics after being involved in a gang fight. His blood pressure is 95/60 mmHg & his pulse is 118/min. Physical exam shows severe penetrating stab wounds to the abdomen. Emergency ultrasound shows intra-abdominal fluid collection. An exploratory laparotomy is performed & the celiac trunk is found to be lacerated. The blood supply to what will be most severely affected by this injury

- *body & tail of the pancreas* Celiac trunk - arises from abdominal aorta - 3 branches = splenic artery, left gastric artery, common hepatic artery splenic artery - courses along superior border of body & tail of pancreas on its way to the spleen --> provides blood supply to *pancreas* - also gives rise to gastric arteries --> *supply fundus of stomach* - left gastro-omental (gastroepiploic) artery --> supplies *greater curvature of stomach* common hepatic artery - supplies the *liver, gallbladder, lesser curvature of the stomach, head of the neck of the pancreas, proximal part of the duodenum*

A 12-year-old girl is brought to the physician because of a severe sore throat. She describes significant pain with swallowing & general feelings of malaise. Her parents report that the patient has had a high fever for the past several days. Physical exam shows an erythematous patch on the upper posterior pharynx & tenderness of the cervical lymph nodes. Her temperature is 39.5 C (103.5 F). The nerve responsible for the patient's throate pain is also involvede in ...

- *sensory input from carotid sinus baroreceptors* - carries general somatic sensation from posterior part of upper pharynx, eustachian tube, middle ear, posterior 1/3 of tongue - sends efferent fibers to stylopharyngeus muscle - carriers parasympathetic fibers that synapse in otic ganglion in pathway from innervation of parotid gland Glossopharyngeal nerve / IX

Symptoms of stroke with Posterior Cerebral Artery

- *visual symptoms* - contralateral hemianopia with macular sparing

3 arteries coming off the celiac trunk

- common hepatic - splenic - left gastric

Recognize pneumoperitoneum

- free air in peritoneal space typically associated with perforation of intestine - CT = criterion standard in assessment of pneumoperitoneum - most common cause = perforated abdominal viscus, generally a perforated peptic ulcer - perforated duodenal ulcer = most common cause of rupture of abdomen

Important portal-caval (systemic) anastomoses

- left gastric vein & esophageal vein of azygos sysem - *superior rectal vein & middle & inferior rectal veins* - paraumbilical veins & radicles of epigastric (superficial & inferior) veins - retrocolic veins & twigs of renal vein

Recognize Superior sagittal sinus thromosis location on cross-section MRI

- often associated with hypercoagulable states - during pregnancy - located in *midsagittal plain along superior aspect of falx cerebri* - drains into confulence of sinuses

Skin of urogenital triangle is innervated by

- pudendal nerve - perineal branches of posterior femoral cutaneous nerve - anterior scrotal or labial branches of ilioinguinal nerve - genital branch of genitofemoral nerve

Branches of gastroduodenal artery

- right gastro-omental artery --> follows greater curvature of stomach - anterior superior pancreaticoduodenal artery = anterior part of head of pancreas, proximal duodenum

Recognize *pneumothorax*

- young white, tall skinny males - can shift heart - causes decreased preload = inadequate perfusion to major organs, like brain - treatment = needle thoracotomy placed in second intercostal space

What type of hernia is acquired (develops after birth)?

Direct hernia - does not enter deep inguinal ring - occurs through posterior wall of inguinal canal - medial to inferior epigastric artery - covered only by peritoneum

Retroperitoneal Structures

A DUCK PEAR Adrenal glands Duodenum (2nd-3rd parts) Ureters Colon (Ascending & Descending) Kidneys Pancreas (everything but the tail) Esophagus Aorta (& IVC) Rectum

Anterior half of external ear canal innervated by

Auriculotemporal nerve

A 61-year-old man with a history of hypertension is brought to the emergency department because of life-sided hemiparesis. Neurologic exam shows dysarthria, and the tongue deviates to the right upon protrusion. There is also a loss of tactile & vibratory sensation over the trunk & over the left upper & lower extremities. An occlusion of what vessels is most likely responsible for the patient's symptoms?

Anterior spinal artery

Anterior interosseous nerve innervation

Branch of median nerve...that supplies... - flexor pollicis longus - half of *flexor digitorum profundus* - *pronator quadratus*

Klumpke palsy

Clumsy palsy = C8-T1

According to TeamSTEPPs model, individuals can learn four primary trainable teamwork skills which are:

Leadership Communication *Situation monitoring* Mutual support - team can enhance 3 types of teamwork outcomes: *performance, knowledge, attitudes* - *teamwork system* designed for healthcare professionals = solution to *improve patient safety within a given organization*

Steeple sign

Croup

Common cause of meningitis in immunocompromised patients, especially in those with a CD4 count of less than 50 cell/mm^3

Cryptococcus neoformans = *monomorphic, encapsulated yeast* CSF findings in case with fungal infection in CNS = 100-1000 cells, mostly lymphocytes - glucose < 45 mm/dL - protein > 50 mg/dL - opening pressures for lumbar puncture in patients with cryptococcal meningitis usually elevated

Runs through the anterior compartment of the leg

Deep fibular (peroneal) nerve - can lead to anterior compartment syndrome = impairment of: dorsiflexion, inversion of foot, extension of toes

Middle-aged miner injured back after accidental explosion. His MRI scan reveals that his spinal cord has shifted to the right because the lateral extension of the pia mater were torn. Function of what structures is most likely impaired?

Denticulate ligament = lateral extension of pia mater

Recognize the right ovary and know its venous drainage

Drainage of both ovaries is to ovarian veins - *right ovarian vein (& testicular vein in male) drains directly to inferior vena cava* - *left ovarian (& testicular) vein drains to left renal vein, which then drains to inferior vena cava*

Difference between Epidural & Subdural hematomas in the way the blood is trapped

Epidural = *does not cross suture lines* Subdural = *crosses suture lines*

*Contains portal triad*

Hepatoduodenal ligament - hepatic artery, portal vein, common bile duct

Connects liver to anterior abdominal wall

Falciform ligament

Principal action of the psoas muscle

Flexes thigh at hip

Ligament that connects lesser curvature of stomach to liver

Gastrohepatic ligament - allows you to access lesser sac during surgery (posterior to stomach)

2 ligaments to be cut when removing spleen

Gastrosplenic - connects greater curvature of stomach to spleen Splenorenal - connects spleen to posterior abdominal wall

Ligament that connects greater curvature of stomach to spleen

Gastrosplenic ligament - one of 2 ligaments to cut when removing spleen (also splenorenal ligament)

CN that lies in tonsillar fossa

Glossopharyngeal nerve (CN IX) - carries sensory & taste info from posterior 1/3 of tongue

CN that lies in tonsillar fossa

Glossopharyngeal nerve (CN IX) - carries sensory & taste info from posterior third of tongue

Lower part of auricle innervated by

Greater auricular nerve

Artery of the rectum & distal third of colon

Inferior mesenteric artery

Gag reflex

Ipsilateral CN IX (afferent) Bilateral vagus nerve (efferent)

Abnormally increased thoracic curvature

Kyphosis - hunchback or humpback

A 55-year-old man with a history of HT is brought to eh ED because of right-sided hemiplegia. His wife states that he has smoked 1 pack of cigarettes daily for the past 35 years & he consumes 3-4 beers daily. He appears to be in some distress & has difficulty speaking. Physical exam shows paralysis of the right side of the mouth, & the right upper & lower extremities. There is no loss of sensation noted on any of his extremities & he is able to wrinkle his forehead on both sides. What lesion is responsible for this patient's symptoms?

Left internal capsule - hemorrhage via uncontrolled HT = right-sided hemiplegia - lower facial paralysis via upper motor neuron lesion

Upper part of auricle innervated by

Lesser occipital nerve

Thoracoacromail trunk branches

PDAC: Pectoral Deltoid Acromial Clavicular

Hirschsprung disease (aganglionic megacolon), dilation of the colon, is caused by absence of what kinds of neural cell bodies?

Parasympathetic *postganglionic* neuron cell bodies - absence of enteric ganglia in lower part of colon - inability to evacuate bowels

Only splanchnic nerves that carry parasympathetic fibers

Pelvic splanchnic nerves / Nervi erigentes - contains preganglionic parasympathetic fibers - responsible for erection

Infarction of this region in one hemisphere can produce homonymous hemianopsia with macular sparing (ex. visual fields reduced on right side in both eyes)

Posterior cerebral artery (PCA) - supplies occipital cortex, where primary visual cortex is located

Most common bacteria associated with Classic acute infective endocarditis in IV drug users / tricuspid insufficiency

Staphylococcus aureus

Structures perforating the diaphragm

T8 = IVC T10 = esophagus, vagus T12 = aorta, thoracic duct, azygos vein

Location of chemoreceptor trigger zone (CTZ), which causes vomiting

area postrema on *floor of fourth ventricle* - metoclopramide = DA antagonist that acts on area postrema - ondansetron

Mediates Ca2+ activation of contractile apparatus in cardiac & skeletal muscle

Troponin (Tn)

Hamate injury -->

Ulnar nerve injury

Erb Palsy

Up palsy = C5-C6

A 46-year-old man comes in with a weakened thenar eminence & complains of not being able to make a fist. He works as a roofer & recently got a divorce. He has smoked one pack per day for the last 25 years. He has not other complaints. What other deficits would you expect as a result of this injury?

Weakness of the 1st & 2nd lumbricals (median nerve)

A 58-year-old man comes to the ED because of numbness in his right face, arm, & leg for the last 2 days. He has HT & has smoked 2 packs of cigarettes per day for 30 years. His temperature is 36.7 C (98.4 F), pulse rate is 79/min, respirations are 16/min, & BP is 202/115 mmHg. Physical exam shows decreased pinprick, temperature, light touch, & vibration sensation over his right face & body. Two-point discrimination is 16 mm on the right index finger & 5 mm on the left. A normal MRI of the brain is shown. These findings are most likely to be caused by a lesion in what labeled structure on a normal MRI?

a = *left ventral posterior thalamus* - decreased sensation over right face & body = all sensory modalities, including those derived from dorsal column & anterolateral system - from contralateral body = VPL - from contralateral face = VPM

Hypertension proximal to coarctation of aorta develops predominately due to

activation of renin-angiotensin-aldosterone system (RAAS) = *decreased juxtaglomerular calcium levels*

Paralysis limited to lower half of face indicates

contralateral UMN lesion

Grown men usually have what type of hernias?

direct

Innervation of glans penis

dorsal artery of penis

Budd-Chiari syndrome, like that seen in Polycythemia vera, best established with

hepatic venography

Genioglossus is innervated by the

hypoglossal nerve (CN XII) - pulls tongue forward (protrudes) & toward opposite side - right genioglossus muscle paralyzed = left genioglossus muscle acts unopposed & pulls tongue forward & to right - tongue "*licks the lesion*" --> tongue deviates to side of lesion

Young boys usually have what type of hernias?

indirect

Paralysis of entire side of face indicates

ipsilateral LMN lesion of CN VII

Where to do a thoracotomy

just above the lower rib to prevent nerve damage

Pectoralis major innercated by

lateral & medial pectoral nerves originating from *lateral* & medial cords of brachial plexus

Cleft lip occurs when ...

lateral maxillary prominence fails to fuse with the medial nasal prominences

Dorsal scapular nerve innervates

levator scapulae rhomboid muscles

Area of body drained by internal iliac/inguinal lymph nodes

lower rectum to anal canal (above pectinate line) *bladder* vagina (middle third) cervix *prostate*

Runs lateral to the tendons of the flexor digitorum superficialism

median nerve - innervates all of the *thenar compartment muscles* at base of the thumb = abductor pollicis brevis, opponens pollicis

Senila cardiac amyloidosis results from

misfolding of wild-type *transthyretin* (formerly called prealbumin) - cardiac amyloidosis in elderly patient, without amyloid deposition in other organs - usually asymptomatic

In a freak hunting accident, 17-year-old boy was shot with an arrow that penetrated into his suboccipital triangle, injuring the suboccipital nerve between the vertebral artery & the posterior arch of the atlas. What muscles would be affected by such a lesion?

muscles of suboccipital area = rectus capitis posterior major = obliquus capitis superior & inferior = semispinalis capitis Suboccipital nerve - derived from dorsal ramus of C1 - emerges between vertebral artery above & posterior arch of atlas below - contains skeletal motor fibers & no cutaneous sensory fibers

What structures would injured after a crushed second cervical vertebrae (axis)?

obliquus capitis inferior alar ligament semispinalis cervicis muscle apical ligament

Cleft palate occurs when...

palatine shelves (maxillary prominences) fail to fuse with each other or the primary palate

Hepatoduodenal ligament holds the

portal triad, hepatic artery, common bile duct

Lies posterior to right carotid sheath

right sympathetic trunk - fibers synapse in superior cervical ganglion on postganglionic sympathetic neurons that innervate structures in head - *dilator pupillae muscle*

Layers of anterior abdominal wall

skin Camper fascia (fatty layer) Scarpa fascia (fibrous layer) external oblique / aponeurosis internal oblique muscle / aponeurosis trasnversus abdominis muscle / aponeurosis transversalis fascia parietal peritoneum

Recognize croup

steeple sign

Splenorenal ligament holds the

tail of the pancreas & splenic vessels

Application of local anesthetic at distal forearm, before median nerve enters the carpal tunnel, is between the

tendons of the palmaris longus and the flexor carpi radialis

Dorsal scapular artery arises directly from

third part of *subclavian artery* - replaces deep (descending) branch of transverse cervical artery

Ligating the uterine artery. Watch out for the

ureter


Related study sets

Module 5: Intracranial Regulation

View Set

Writing Point-Slope Form Equations from Two Points

View Set

Anthropology 250 Practice Final - UVic 2017 Fall

View Set

Revature Panel Interview Questions

View Set

Psychiatric-Mental Health Practice Exam HESI

View Set

MI Life Insurance Certificate Chapter 1

View Set

Chapter 13 Administration and Risk Management Final

View Set

CSE 643 - Computer Security - SQL Injection

View Set

Saunders Unit II Issues in Nursing Q 13-25

View Set

Educational Psychology Final Study Guide

View Set

Fundamentals of Nursing - Ch 23 Legal Implications in Nursing Practice

View Set